Đến nội dung

Hình ảnh

ĐỀ THI VÒNG 1+VÒNG 2 MÔN TOÁN TUYỂN SINH VÀO LỚP $10$ THPT CHUYÊN KHOA HỌC TỰ NHIÊN NĂM 2015-2016


  • Please log in to reply
Chủ đề này có 81 trả lời

#61
hoangtunglam

hoangtunglam

    Binh nhất

  • Thành viên
  • 25 Bài viết

phải rồi



#62
viet nam in my heart

viet nam in my heart

    Thượng sĩ

  • Điều hành viên OLYMPIC
  • 242 Bài viết

Loiw

 

Lời giải câu cuối nè :

Giả sử trong $2015$ điểm có $n$ điểm thẳng hàng ($n\leq2014$) (Có thể có nhiều điểm thẳng hàng khác nhưng không thẳng hàng với $n$ điểm này nhưng không cần quan tâm)

Với $n=1$ tức là không có bất kì hai điểm nào thẳng hàng thì điều phải chứng minh đã rõ

Với $n>1$ thì số đường kẻ từ các điểm còn lại đến $n$ điểm này là $n(2015-n)$

Cộng thêm 1 đường thẳng nữa đi qua n điểm thì số đường thẳng ít nhất là $n(2015-n)+1$

Giờ chỉ việc chứng minh nó $\geq2015$ thôi. Thật vậy ta có : $2015(n-1)-(n-1)(n+1)\geq0<=>(2014-n)(n-1)\geq0$ (Điều này luôn đúng)

Vậy ta có điều phải chứng minh

Lời giải này sai rồi bạn. $n(2015-n)$ đường thẳng của bạn có thể trùng nhau  :(


Bài viết đã được chỉnh sửa nội dung bởi viet nam in my heart: 01-06-2015 - 22:32

"Nếu bạn hỏi một người giỏi trượt băng làm sao để thành công, anh ta sẽ nói với bạn: ngã, đứng dậy là thành công." Isaac Newton

VMF's Marathon Hình học Olympic


#63
ZzNightWalkerZz

ZzNightWalkerZz

    Trung sĩ

  • Thành viên
  • 159 Bài viết

Lời giải này sai rồi bạn. $n(2015-n)$ đường thẳng của bạn có thể trùng nhau  :(

Bạn có nhầm lẫn không vậy ? Đã nói là các điểm còn lại không hề thẳng hàng với n điểm này nên không thể nào trùng nhau


.

Reaper

.

.

The god of carnage


#64
viet nam in my heart

viet nam in my heart

    Thượng sĩ

  • Điều hành viên OLYMPIC
  • 242 Bài viết

Bạn có nhầm lẫn không vậy ? Đã nói là các điểm còn lại không hề thẳng hàng với n điểm này nên không thể nào trùng nhau

Đúng là các điểm còn lại không hề thẳng hàng với n điểm này.

Nhưng rõ ràng lấy 1 điểm trong n điểm đó ra thì có thể thẳng hàng với các điểm còn lại ( trường hợp này thì bạn tính sao ) 


"Nếu bạn hỏi một người giỏi trượt băng làm sao để thành công, anh ta sẽ nói với bạn: ngã, đứng dậy là thành công." Isaac Newton

VMF's Marathon Hình học Olympic


#65
dogsteven

dogsteven

    Đại úy

  • Thành viên
  • 1567 Bài viết

Bất đẳng thức vòng 1.

$ab+bc+ca+abc\leqslant 4\Leftrightarrow \dfrac{a}{a+2}+\dfrac{b}{b+2}+\dfrac{c}{c+2}\leqslant 1$

Áp dụng bất đẳng thức Cauchy-Schwarz: $\sum a(a+2) \geqslant \sum \dfrac{a}{a+2}.\sum a(a+2)\geqslant (a+b+c)^2$

$\Leftrightarrow a+b+c\geqslant ab+bc+ca$ mà $a^2+b^2+c^2\geqslant ab+bc+ca$ nên $a^2+b^2+c^2+a+b+c\geqslant 2(ab+bc+ca)$


Quyết tâm off dài dài cày hình, số, tổ, rời rạc.


#66
huuhieuht

huuhieuht

    Trung sĩ

  • Thành viên
  • 191 Bài viết

BÌNH LUẬN BÀI CUỐI TOÁN CHUYÊN KHTN
Lời giải: 
(1) Trong 2015 điểm thuộc S có nhiều nhất 2014 điểm thẳng hàng. Khi đó số đường thẳng phân biệt được tạo thành đi qua 2 điểm thuộc S là ít nhất.(xét trường hợp còn lại thì quá rõ để cm lớn hơn rùi hay đó là điều hiển nhiên)
(2) Từ 1 điểm khác 2014 điểm thẳng hàng luôn kẻ được 2014 đường thẳng phân biệt.
Từ (1) và (2) ta có ít nhất 2015 đường thẳng phân biệt được tạo thành đi qua ít nhất 2 điểm thuộc S.

Mình hơi giật mình khi đọc câu cuối. Thật không thể tin được Biểu tượng cảm xúc colonthree nó bị dễ 


Bài viết đã được chỉnh sửa nội dung bởi huuhieuht: 03-06-2015 - 22:53

Không có giới hạn tư duy nào của con người ngoài giới hạn do chính con người đặt ra (Napoleon Hill)   :D  :D  :D  :like  ~O) 


#67
vda2000

vda2000

    Sĩ quan

  • Thành viên
  • 301 Bài viết

 

 

BÌNH LUẬN BÀI CUỐI TOÁN CHUYÊN KHTN
Lời giải: 
(1) Trong 2015 điểm thuộc S có nhiều nhất 2014 điểm thẳng hàng. Khi đó số đường thẳng phân biệt được tạo thành đi qua 2 điểm thuộc S là ít nhất.(xét trường hợp còn lại thì quá rõ để cm lớn hơn rùi hay đó là điều hiển nhiên)
(2) Từ 1 điểm khác 2014 điểm thẳng hàng luôn kẻ được 2014 đường thẳng phân biệt.
Từ (1) và (2) ta có ít nhất 2015 đường thẳng phân biệt được tạo thành đi qua ít nhất 2 điểm thuộc S.

Mình hơi giật mình khi đọc câu cuối. Thật không thể tin được Biểu tượng cảm xúc colonthree nó bị dễ 

 

Là sao vậy?


Bài viết đã được chỉnh sửa nội dung bởi vda2000: 04-06-2015 - 14:32

$\boxed{\textrm{Silence is the peak of contempt!}}$

If you see this, you will visit my facebook.....!


#68
quanghung86

quanghung86

    Thiếu úy

  • Điều hành viên
  • 632 Bài viết

Có thể tham khảo lời giải và bình luận hai bài hình học tại đây http://analgeomatica...op-10-thpt.html



#69
duypro154

duypro154

    Trung sĩ

  • Thành viên
  • 133 Bài viết

1. 

 

$x-1|xy-1\Leftrightarrow x-1|y(x-1)+y-1\Leftrightarrow x-1|y-1$ Tương tự, $y-1|x-1$ suy ra $x-1=y-1$ hay $x=y$

 

Khi đó $(x-1)^2|x^2-1\Leftrightarrow x-1|x+1$ hay $x-1|2\rightarrow$ ...

 

2. $P^2=\frac{-2y-1}{(3y+1)^2}$ $\Leftrightarrow (9P^2)y^2+y(6P^2+2)+P^2+1=0$

 

Pt có nghiệm khi $(3P^2+1)^2-9P^2(P^2+1)\geq 0\Leftrightarrow 1\geq 3P^2\Leftrightarrow -\sqrt{\frac{1}{3}}\leq P\leq \sqrt{\frac{1}{3}}$

Bài chia hết bạn có thể làm rõ hơn k? Mình k hiểu lắm



#70
XanCao

XanCao

    Binh nhì

  • Thành viên
  • 12 Bài viết

Câu III

 

a) Dễ thấy $F\in AB$ và $E\in AC$

 

Có $BF=BD\Rightarrow \frac{BF}{AB}=\frac{BD}{AB}$. Tương tự $\frac{CE}{AC}=\frac{CD}{AC}$

 

Mà $\frac{BD}{AB}=\frac{CD}{AC}\Rightarrow \frac{BF}{AB}=\frac{CE}{AC}\Rightarrow EF\parallel BC$

 

b) Trước tiên có $NJ\parallel DE,MJ\parallel FD$

 

Từ phần a) có $\angle BFD=\angle DFE$ và $\angle CED=\angle DEF$

 

Tứ giác $FNPA$ và $APME$ nội tiếp nên $\angle NPM=\angle NPA+\angle MPA$

                           

                                 $=\angle BFD+\angle DEC=\angle DFE+\angle DEF=\angle EJM+\angle FJN=180^0-\angle NJM$ 

 

Suy ra $NJMP$ nội tiếp

 

c) Từ phần b) do $NJMP$ nội tiếp nên

 

$\angle NPJ=\angle NMJ=\angle MJE=\angle DFE=\angle BFD=\angle NPA$ ( do $FNPA$ nội tiếp)

 

Do đó $\overline{P,J,A}$ ( đpcm)

 

Làm sao c/m được tứ giác FNPA nội tiếp ạ?


With many people, mathematics is just a subject...

With me, it's my passion...

:icon12:Mathematics  :icon12: 

 


#71
Quoc Tuan Qbdh

Quoc Tuan Qbdh

    DragonBoy

  • Điều hành viên THCS
  • 1005 Bài viết

#Lề : Đề trên mạng : Câu $2b$    $x^{2}y^{2}$ --> $x^{3}y^{3}$

:mellow:  :mellow:  :mellow:  :mellow:  



#72
Dinh Xuan Hung

Dinh Xuan Hung

    Thành viên nổi bật 2015

  • Thành viên nổi bật 2016
  • 1396 Bài viết

           ĐẠI HỌC KHOA HỌC TỰ NHIÊN                                         ĐỀ THI TUYỂN SINH VÀO LỚP 10

           THPT KHOA HỌC TỰ NHIÊN                                                THPT CHUYÊN NĂM 2015-2016

                                                                                                                  Môn:Toán (Vòng 1)

                                                                                        Thời gian làm bài 120 phút (Không kể thời gian giao đề)

        ĐỀ CHÍNH THỨC

   

Câu IV.(1 điểm)

2)Cho $a,b,c$ là các số thực dương thỏa mãn $ab+bc+ac+abc\leq 4$.Chứng minh rằng:$a^2+b^2+c^2+a+b+c\geq 2(ab+bc+ac)$

                                                                 

Cán bộ coi thi không giải thích gì thêm

Sau rất nhiều ngày suy nghĩ em có một cách khác nếu nó không hay thì mọi người từng ném đá nhé :))

Đầu tiên dễ có:$a^2+b^2+c^2\geq ab+bc+ac$ nên ta cần chứng minh $a+b+c\geq ab+bc+ac$

Vì vai trò của $a,b,c$ là như nhau nên giả sử:$a\geq b\geq c> 0$

Ta có:$3c+c^3\leq a+b+c+abc\leq 4\Leftrightarrow c^3+3c-4\leq 0\Leftrightarrow c\leq 1$

  • Nếu $1\geq a\geq b\geq c> 0$

$\Rightarrow \left\{\begin{matrix} ab\leq a & & & \\ bc\leq b & & & \\ ca\leq c & & & \end{matrix}\right. \Rightarrow a+b+c\geq ab+bc+ac$

  • Nếu $a\geq b\geq 1\geq c>0$

Ta có:$4\geq a+b+c+abc> a+b\geq 2\sqrt{ab}\Rightarrow \sqrt{ab}< 2\Leftrightarrow ab< 4$

Do đó:$(a+b-2)^2\geq 4(a-1)(b-1)\geq ab(a-1)(b-1)\Leftrightarrow (a+b-ab)(ab+1)\geq (4-a-b)(a+b-1 )\Leftrightarrow a+b-ab\geq \frac{4-a-b}{ab+1}(a+b-1)\geq c(a+b-1)\Leftrightarrow a+b+c\geq ab+bc+ac$

  • Nếu $a\geq 1\geq b\geq c>0$

Ta có:$(a-1)(b-1)(c-1)\geq 0\Rightarrow a+b+c\geq ab+bc+ac+1-abc(2)$

Áp dụng BĐT AM-GM:

$4\geq a+b+c+abc\geq 4\sqrt[4]{(abc)^2}\Rightarrow abc\leq 1$

Kết hợp với $(2)$ suy ra điều phải chứng minh

Đẳng thức xảy ra khi $a=b=c=1$


Bài viết đã được chỉnh sửa nội dung bởi Dinh Xuan Hung: 18-06-2015 - 11:07


#73
dogsteven

dogsteven

    Đại úy

  • Thành viên
  • 1567 Bài viết

Sau rất nhiều ngày suy nghĩ em có một cách khác nếu nó không hay thì mọi người từng ném đá nhé :))

Đầu tiên dễ có:$a^2+b^2+c^2\geq ab+bc+ac$ nên ta cần chứng minh $a+b+c\geq ab+bc+ac$

Vì vai trò của $a,b,c$ là như nhau nên giả sử:$a\geq b\geq c> 0$

Ta có:$3c+c^3\leq a+b+c+abc\leq 4\Leftrightarrow c^3+3c-4\leq 0\Leftrightarrow c\leq 1$

  • Nếu $1\geq a\geq b\geq c> 0$

$\Rightarrow \left\{\begin{matrix} ab\leq a & & & \\ bc\leq b & & & \\ ca\leq c & & & \end{matrix}\right. \Rightarrow a+b+c\geq ab+bc+ac$

  • Nếu $a\geq b\geq 1\geq c>0$

Ta có:$4\geq a+b+c+abc> a+b\geq 2\sqrt{ab}\Rightarrow \sqrt{ab}< 2\Leftrightarrow ab< 4$

Do đó:$(a+b-2)^2\geq 4(a-1)(b-1)\geq ab(a-1)(b-1)\Leftrightarrow (a+b-ab)(ab+1)\geq (4-a-b)(a+b-1 )\Leftrightarrow a+b-ab\geq \frac{4-a-b}{ab+1}(a+b-1)\geq c(a+b-1)\Leftrightarrow a+b+c\geq ab+bc+ac$

  • Nếu $a\geq 1\geq b\geq c>0$

Ta có:$(a-1)(b-1)(c-1)\geq 0\Rightarrow a+b+c\geq ab+bc+ac+1-abc(2)$

Áp dụng BĐT AM-GM:

$4\geq a+b+c+abc\geq 4\sqrt[4]{(abc)^2}\Rightarrow abc\leq 1$

Kết hợp với $(2)$ suy ra điều phải chứng minh

Đẳng thức xảy ra khi $a=b=c=1$

Muốn ném đã dòng đầu -_-


Quyết tâm off dài dài cày hình, số, tổ, rời rạc.


#74
dogsteven

dogsteven

    Đại úy

  • Thành viên
  • 1567 Bài viết

Sau rất nhiều ngày suy nghĩ em có một cách khác nếu nó không hay thì mọi người từng ném đá nhé :))

Đầu tiên dễ có:$a^2+b^2+c^2\geq ab+bc+ac$ nên ta cần chứng minh $a+b+c\geq ab+bc+ac$

Vì vai trò của $a,b,c$ là như nhau nên giả sử:$a\geq b\geq c> 0$

Ta có:$3c+c^3\leq a+b+c+abc\leq 4\Leftrightarrow c^3+3c-4\leq 0\Leftrightarrow c\leq 1$

  • Nếu $1\geq a\geq b\geq c> 0$

$\Rightarrow \left\{\begin{matrix} ab\leq a & & & \\ bc\leq b & & & \\ ca\leq c & & & \end{matrix}\right. \Rightarrow a+b+c\geq ab+bc+ac$

  • Nếu $a\geq b\geq 1\geq c>0$

Ta có:$4\geq a+b+c+abc> a+b\geq 2\sqrt{ab}\Rightarrow \sqrt{ab}< 2\Leftrightarrow ab< 4$

Do đó:$(a+b-2)^2\geq 4(a-1)(b-1)\geq ab(a-1)(b-1)\Leftrightarrow (a+b-ab)(ab+1)\geq (4-a-b)(a+b-1 )\Leftrightarrow a+b-ab\geq \frac{4-a-b}{ab+1}(a+b-1)\geq c(a+b-1)\Leftrightarrow a+b+c\geq ab+bc+ac$

  • Nếu $a\geq 1\geq b\geq c>0$

Ta có:$(a-1)(b-1)(c-1)\geq 0\Rightarrow a+b+c\geq ab+bc+ac+1-abc(2)$

Áp dụng BĐT AM-GM:

$4\geq a+b+c+abc\geq 4\sqrt[4]{(abc)^2}\Rightarrow abc\leq 1$

Kết hợp với $(2)$ suy ra điều phải chứng minh

Đẳng thức xảy ra khi $a=b=c=1$

Đã thấy một lỗi sai ở chỗ màu đỏ. Với $a=4, b=1, c=0$ thì bất đẳng thức này hoàn toàn bị ngược.

Mà cũng dễ nhận ra, nếu đây là một bất đẳng thức hệ quả thì có ngay $a+b+c\leqslant ab+bc+ca$ vô lý.

Tuy nhiên, điều này phù hợp nếu ta xét $4\geqslant a+b+c+abc$ và $4\leqslant a+b+c+abc$


Bài viết đã được chỉnh sửa nội dung bởi dogsteven: 23-06-2015 - 21:00

Quyết tâm off dài dài cày hình, số, tổ, rời rạc.


#75
Dinh Xuan Hung

Dinh Xuan Hung

    Thành viên nổi bật 2015

  • Thành viên nổi bật 2016
  • 1396 Bài viết

Đã thấy một lỗi sai ở chỗ màu đỏ. Với $a=4, b=1, c=0$ thì bất đẳng thức này hoàn toàn bị ngược.

Mà cũng dễ nhận ra, nếu đây là một bất đẳng thức hệ quả thì có ngay $a+b+c\leqslant ab+bc+ca$ vô lý.

Tuy nhiên, điều này phù hợp nếu ta xét $4\geqslant a+b+c+abc$ và $4\leqslant a+b+c+abc$

Mình tưởng đề cho $a,b,c$ dương thỏa mãn $4\geq a+b+c+abc$ từ đầu mà  :closedeyes:



#76
dogsteven

dogsteven

    Đại úy

  • Thành viên
  • 1567 Bài viết

Mình tưởng đề cho $a,b,c$ dương thỏa mãn $4\geq a+b+c+abc$ từ đầu mà  :closedeyes:

Thánh đọc đề là đây =)) Đề cho $ab+bc+ca+abc\leqslant 4$ chứ đâu có $a+b+c+abc\leqslant 4$

Mà theo $a+b+c+abc\leqslant 4$ cũng được.


Quyết tâm off dài dài cày hình, số, tổ, rời rạc.


#77
nloan2k1

nloan2k1

    Thượng sĩ

  • Thành viên
  • 219 Bài viết

Giả sử $x$ nằm trên đường chéo thứ $k$ thì trước nó là $k-1$ đường chéo, mà đường chéo thứ $i$ có $i$ số hạng nên

$1+2+...+(k-1)< x\le 1+2+...+k$ $\Leftrightarrow \dfrac{k(k-1)}{2}< x\le \dfrac{k(k+1)}{2}\Leftrightarrow\begin{cases}k^2-k-2x< 0\\ k^2+k-2x\ge 0 \end{cases}$

$\Rightarrow \dfrac{-1+\sqrt{1+8x}}{2}\le k<\dfrac{1+\sqrt{1+8x}}{2}\Rightarrow k=\left\lceil\dfrac{-1+\sqrt{1+8x}}{2}\right\rceil$

Phải không nào?

Thầy cho em hỏi sao lại có đoạn này ạ?

Em vẫn chưa hiểu cho lắm.

$x$ nằm trên đường chéo thì có liên quan gì đến $1+2+...+(k-1)$ và $1++2+...+k$ há thầy? 

Nếu thử vào thì đúng ơ nhưng mà tại sao ạ? :'( 


Bài viết đã được chỉnh sửa nội dung bởi nloan2k1: 18-11-2015 - 21:19


#78
hxthanh

hxthanh

    Tín đồ $\sum$

  • Hiệp sỹ
  • 3915 Bài viết
$x$ là chỉ số là vị trí số hạng cần tìm. Nó nằm trên đường chéo thứ $k$ thì phía trước nó có:
1 số hạng ở đường chéo thứ nhất
2 số hạng ở đường chéo thứ hai
...
$k-1$ số hạng ở đường chéo thứ $k-1$
Như vậy vị trí $x$ phải lớn hơn tổng số các số hạng này

#79
nloan2k1

nloan2k1

    Thượng sĩ

  • Thành viên
  • 219 Bài viết

$x$ là chỉ số là vị trí số hạng cần tìm. Nó nằm trên đường chéo thứ $k$ thì phía trước nó có:
1 số hạng ở đường chéo thứ nhất
2 số hạng ở đường chéo thứ hai
...
$k-1$ số hạng ở đường chéo thứ $k-1$

Như vậy vị trí $x$ phải lớn hơn tổng số các số hạng này

 Em thấy điều này chỉ đúng khi các đường chéo $k$ nằm trong nửa ô vuông đổ lại thôi ạ, vậy thì ta lại phải chứng minh nữa có phải không ạ? 

Em vẫn chưa hiểu cứ sao $x$ lại phải lớn hơn tổng các số hạng này thầy ơiiii thầy giúp emm 



#80
doanminhhien127

doanminhhien127

    Trung sĩ

  • Thành viên
  • 136 Bài viết

Câu I.1

a)

$a^{2}+3a=b^{2}+3b$

$\Leftrightarrow a^{2}-b^{2}+3a-3b=0$

$\Leftrightarrow (a-b)(a+b)+3(a-b)=0$

$\Leftrightarrow (a-b)(a+b+3)=0$

$\Leftrightarrow a-b=0$ hoặc $a+b+3=0$

$\Leftrightarrow a=b$ (0 TMĐK) hoặc $a+b=-3$

Vậy $a+b=-3$ (đpcm)

b) Ta có:

$a^{2}+3a=b^{2}+3b=2$

$\Rightarrow a^{2}+b^{2}+3a+3b=4$

$\Leftrightarrow a^{2}+b^{2}=4-3(a+b)$

$\Leftrightarrow a^{2}+b^{2}=13$

$(a+b)^{2}=9 \Leftrightarrow a^{2}+b^{2}+2ab=9 \Leftrightarrow 2ab=9-(a^{2}+b^{2}) \Leftrightarrow ab=-2$

$a^{3}+b^{3}=(a+b)(a^{2}-ab+b^{2})=(-3)(13+2)=-45$ (đpcm)

Cho mình hỏi : Tại sao ở câu I.1a)$a=b$(0 TMĐK). Vậy ĐK của bạn là gì?

Ở câu b) Tại sao $\Rightarrow a^{2}+b^{2}+3a+3b=4$


Mong các bạn có thể giải bài giúp mình càng sớm, chi tiết dễ hiểu ( nhiều cách khác nhau) càng tốt. Cảm ơn nhiều.  





1 người đang xem chủ đề

0 thành viên, 1 khách, 0 thành viên ẩn danh